Monday 16 February 2015

analysis - $f: [0,infty) to mathbb{R}$ is continuous and $displaystyle lim_{x to infty}f(x) = L < infty$. Prove that $f$ is uniformly continuous.



Suppose that $f: [0,\infty) \to \mathbb{R}$ is a continuous function and $\displaystyle \lim_{x \to \infty}f(x)$ exists. Prove that $f$ is uniformly continuous on $[0,\infty)$.



Here's my solution, but because the problem has been given in a math contest preparation program I'm afraid that there's some point that I might be missing.



Since $\displaystyle \lim_{x \to \infty}f(x)$ exists, there exists $L \in \mathbb{R}$ such that:



$\forall \epsilon>0, \exists M>0 \text{ such that } x > M \implies |f(x)-L|<\epsilon/2$




$\forall \epsilon>0, \exists M>0 \text{ such that } y > M \implies |f(y)-L|<\epsilon/2$



Therefore, $\forall x,y \in (M,\infty)$ we have shown that:
$|f(x)-f(y)|<\epsilon$.



Now, let's consider $f$ on $[0,M]$. Since $[0,M]$ is a compact interval and $f$ is continuous, $f$ is uniformly continuous on $[0,M]$:



$\forall \epsilon>0, \exists \delta_1>0, \forall x,y \in [0,M]: |x-y|<\delta_1 \implies |f(x)-f(y)|<\epsilon$.




Now let us focus our attention at $x=M$.



Since $f: [0,\infty) \to \mathbb{R}$ is continuous, it is continuous at $x=M$ as well. Therefore:



$\epsilon>0, \delta_2>0, \forall x: |x-M|<\delta_2 \implies |f(x)-f(M)|<\epsilon/2$



So, for any $x,y \in (M-2\delta_2,M+2\delta_2)$ we have $|f(x)-f(y)|<\epsilon$



Now if we set $\delta=\min\{\delta_1,\delta_2\}$ we see that all the $3$ cases above become true and we conclude that $\forall x,y \in [0,\infty): |x-y|<\delta \implies |f(x)-f(y)|<\epsilon$. This proves that $f$ is uniformly continuous on $[0,\infty)$.




Is my nonsense considered a proof?


Answer



Did you make a mistake in ".. for any $x, y \in (x-2\delta_2, x+2\delta_2)$..."? Because M'd disappeared. But I think that you are almost there.



It worth trying this with sequence. If the result is false, then there is $\epsilon>0$ and sequences $x_n$ and $y_n$ such that



$$|x_n - y_n| < 1/n$$
and
$$|f(x_n)-f(y_n)|\geq \epsilon.$$




The hypothesis shows that $y_n$ and $x_n$ are bounded, and Bolzano Weierstrass gives a contradiction.


No comments:

Post a Comment

real analysis - How to find $lim_{hrightarrow 0}frac{sin(ha)}{h}$

How to find $\lim_{h\rightarrow 0}\frac{\sin(ha)}{h}$ without lhopital rule? I know when I use lhopital I easy get $$ \lim_{h\rightarrow 0}...